LSAT and Law School Admissions Forum

Get expert LSAT preparation and law school admissions advice from PowerScore Test Preparation.

User avatar
 sunshine123
  • Posts: 44
  • Joined: Jul 18, 2022
|
#96908
Perfect, thank you for your time!
User avatar
 sunshine123
  • Posts: 44
  • Joined: Jul 18, 2022
|
#96916
Hello,

One last question. I'm leaving the question with the following understanding: On such questions, we need to Advance P's position while also responding to, by countering, Q's position. More precisely, we advance the CONCLUSION of P while countering the CONCLUSION of Q. Now, answer choice A seems to be a valid counter to Q's position because it entails a benefit of computer training, namely the ability to deal with TODAYS world. However, though it counters the conclusion of Q, it does not really advance P's position, since P's position is about TOMORROWS world. Frustratingly, A is NOT at odds with P's position, it simply does not propel it forward.

Does that sound right? Thanks in advance
User avatar
 sunshine123
  • Posts: 44
  • Joined: Jul 18, 2022
|
#96983
Or, rather, it does not propel the position of P because it speaks of general training as opposed to early, kindergarden training. Regardless, my observation is still that A counters Q's argument without advancing P's argument. Is that so?
User avatar
 sunshine123
  • Posts: 44
  • Joined: Jul 18, 2022
|
#97151
Hello,

One last question. I'm leaving the question with the following understanding: On such questions, we need to Advance P's position while also responding to, by countering, Q's position. More precisely, we advance the CONCLUSION of P while countering the CONCLUSION of Q. Now, answer choice A seems to be a valid counter to Q's position because it entails a benefit of computer training, namely the ability to deal with TODAYS world. However, though it counters the conclusion of Q, it does not really advance P's position, since P's position is about TOMORROWS world AND about EARLY KINDERGARDEN training as opposed to general training. Frustratingly, A is NOT at odds with P's position, it simply does not propel it forward.

Does that sound right? Thanks in advance
User avatar
 lsatquestions
  • Posts: 66
  • Joined: Nov 08, 2021
|
#97221
lsatquestions wrote: Fri Aug 12, 2022 12:37 pm
Nikki Siclunov wrote: Sat Aug 27, 2011 4:44 pm Answer choice (A) is incorrect, because parent P never suggested that children receive "regular training" in computers or computer languages. Her point is that computer training be introduced as early as possible, not as regularly as possible. Since answer choice (A) would not support parent P's argument, it is incorrect.

Answer choice (B) is also incorrect. Parent Q's argument is that teaching kindergartners how to use computers would simply be pointless; she never suggested that they would be incapable of adapting to new technologies. Even if answer choice (B) were true and today's children are perfectly capable of adapting to use new technologies, parent Q would still have a point arguing that introducing kindergartners to computers would be a waste of time. Since answer choice (B) does not weaken parent Q's argument, it will not provide a strong logical counter for parent P.

Hi Nikki,

Parent P says that computers should be introduced in Kindergarten, but he/she also argues that computer languages should be taught in high school. Early introduction is just one part of the argument. Do you mind further explaining why answer A) is incorrect? Is it that we can't equate regular training with learning computer languages in high school - if so why not? Or does it have to do with the later part of the answer choice about dealing with society?
bump
 Robert Carroll
PowerScore Staff
  • PowerScore Staff
  • Posts: 1819
  • Joined: Dec 06, 2013
|
#97235
sunshine,

Answer choice (A) doesn't seem to counter Q because saying that regular training is necessary doesn't do anything to show that the training will actually be effective. Q has already objected that the particular computer skills introduced to children and students might not be effective because of changing technology. Saying that regular training is necessary doesn't really defuse this objection - Q can easily say "But I already showed that the particular kind of regular training you're talking about won't work." It's as if P said that people need to learn to ride horses, Q said that automobiles are becoming more current, so training with horses is not relevant, and P responds "But people will need regular training to be able to operate whatever mode of transportation is going to exist in the future." Sure, but it makes a big difference what that training is, if one person is objecting that the specific training being suggested won't actually do any good.

So I think answer choice (A) fails to counter Q's objection.

lsatquestions,

The above should be a fine way to show that answer choice (A) is wrong and thus reply to your post as well!

Robert Carroll

Get the most out of your LSAT Prep Plus subscription.

Analyze and track your performance with our Testing and Analytics Package.